当前位置:文档之家› 组合问题------陶平生

组合问题------陶平生

组合问题------陶平生
组合问题------陶平生

组合问题-A 解答(陶平生)

1、将数集},...,,{21n a a a A =中所有元素的算术平均值记为)(A P ,(n

a a a A P n +++=...)(21). 若B 是A 的非空子集,且)()(A P B P =,则称B 是A 的一个“均衡子集”.

试求数集}9,8,7,6,5,4,3,2,1{=M 的所有“均衡子集”的个数.

解:由于()5P M =,令{}{}54,3,2,1,0,1,2,3,4M x x M '=-∈=

----,则()0P M '=, 依照此平移关系,M 和M '的均衡子集可一一对应.

用()f k 表示M '的k 元均衡子集的个数,显然有(9)(1)1f f ==(M '的9元均衡子集只有M ',一元均衡子集只有{}0).

M '的二元均衡子集共四个,为{,},1,2,3,4i B i i i =-=, 因此(2)4f =.

M '的三元均衡子集有两种情况:

(1)含有元素0的为{0}{,0,},1,2,3,4i B i i i =-= , 共四个;

(2)不含元素0的,由于等式312,413=+=+可表示为3120,3120-++=--=以及4130,4130-++=--=,得到4个均衡子集{3,1,2},{3,1,2},{4,1,3},{4,1,3}------,因此(3)448f =+=.

M '的四元均衡子集有三种情况:

(1)每两个二元均衡子集之并:,14i j B B i j ≤<≤ , 共6个集;

(2)不含元素0的三元均衡子集与{}0的并集,共4个集;

(3)以上两种情况之外者,由于等式1423+=+可表为14230--++=以及14230+--=得2个均衡子集{1,4,2,3}--与{1,4,2,3}--,因此()464212f =++=.

又注意到,除M '本身外,若B '是M '的均衡子集,当且仅当其补集''M C B 也是M '的均衡子集,二者一一对应. 因此(9)(),1,2,3,4f k f k k -==.

从而M '的均衡子集个数为94

11()(9)2()12(14812)51k k f k f f k ===+=++++=∑∑.

即M 的均衡子集有51个.

2、若集合{}1,2,,200M = 的子集A 中的每个元素都可表为两个自然数(允许相同)的平方和,求这种集A 中元素个数的最大值.

解:不超过200的平方数是 22220,1,2,,14 .

显然,2221,2,,14 中的每个数2k 可表为22

0k +形式,这种数共有14个;

而2221,2,,10 中的每一对数(允许相同)的和在M 中,这种数有2101055C +=个,

(其中,22x x +形式的数10个,()22 x y x y +≠形式的数210C 个). 其次,()2211 1,2,,8x x += 形式的数8个;()22

12 1,2,,7x x += 形式的数7个; ()2213 1,2,,5x x += 形式的数5个;()2214 1,2x x +=形式的数2个;共得22个,

再考虑重复情况,利用如下事实:若()2222

, ,, ,x a b y c d a b c d =+=+≠≠则 ()()()()2222

xy ac bd ad bc ac bd ad bc =++-=-++

不超过40且能表为两个不同正整数的平方和的数有 5,10,13,17,20,25,26,29,34,37,40,该组中的每个数与5的积,以及213都在集M 中,且都可用两种方式表为平方和,故各被计算了两次,累计有12次重复(10,13,17,20与10的积已包含在以上乘积组中);因此,集A 中元素个数的最大值为

1455221279++-=.

3、将前九个正整数1,2,,9 分成三组,每组三个数,使得每组中的三数之和皆为质数;求出所有不同分法的种数.

证:()

01、由于在1,2,,9 中,三个不同的数之和介于6和24之间,其中的质数有7,11,13,17,19,23这六个数,今将这六数按被3除的余数情况分为两类:

{}7,13,19A =,其中每个数被3除余1;{}11,17,23B =,其中每个数被3除余2;

假若所分成的,,A B C 三组数对应的和,,a b c p p p 为互异质数,则因

12945a b c p p p ++=+++= 被3整除,

故三个和数,,a b c p p p 必为同一类数,因为A 类三数和713193945++=<,B 类三数和1117235145++=>,矛盾!

故三个和数中必有两个相等.

()02、据()0

1知,将45表成7,11,13,17,19,23中的三数和(其中有两数相等),只有四

种情况:()119197++;()2171711++;()3131319++;()4111123++.

由于在1,2,,9 中有5个奇数,故分成的三组中必有一组,三数全为奇数,另两组各有一个奇数.

对于情形()1,和为7的组只有{}1,2,4,剩下六数3,5,6,7,8,9,分为和为19的两组,且其中一组全为奇数,只有唯一的分法:{}3,7,9与{}5,6,8;

对于情形()2,若三奇数的组为{}1,7,9,则另两组为 {}{}4,5,8,2,3,6;或

{}{}3,6,8,2,4,5;

若三奇数的组为{}3,5,9,则另两组为 {}{}2,8,7,1,4,6,或{}{}4,6,7,1,2,8; 若三奇数的组为{}1,3,7,则另两组为 {}{}2,6,9,4,5,8;共得分法5种;

对于情形()3,若三奇数的组为{}3,7,9,则另两组为 {}{}1,4,8,2,5,6;

若三奇数的组为{}1,3,9,则另两组为 {}{}2,4,7,5,6,8或{}{}2,5,6,4,7,8;

若三奇数的组为{}1,5,7,则另两组为 {}{}3,4,6,2,8,9或{}{}2,3,8,4,6,9;

共得分法5种;

对于情形()4,和为23的组只有{}6,8,9,则另两组为 {}{}1,3,7,2,4,5;

据以上,共计得到155112+++=种分法.

4、设{}1,2,,17M = ,若有四个互异数,,,a b c d M ∈,使得()mod17a b c d +≡+,就称{},a b 与{},c d 是集M 的一个“平衡对”,求集M 中“平衡对”的个数.

解:将圆周17等分,其分点按顺时针方向顺次记为1217,,,A A A ,则

()mod17m n k l +≡+当且仅当弦m n A A ∥k l A A .

注意如下事实:圆周17等分点的任一对分点连线都不是直径,因

此全部弦共有17个方向(分别与过i A 的切线平行,1,2,,17i = ).

与过i A 的切线平行的弦有8条,共形成2828C =个“平行弦对”,若考虑所有17个方向,共得 2817476?=个“平行弦对”.即M 中

有476个平衡对.

5、某校有2009名新生,每人至少认识其中n 人,试求n 的最小值,使得其中必存在彼此认识的8个人.

解:记这2009个人的集合为{}122009,,,M v v v = ,i v 所认识的人的集合记为, 1,2,2009i A i = ,则i A n ≥,且 1,2,2009i i v A i ?= ,

若12,v v 是M 中相识的两人,则有121222009A A A A A B n =+-≥- ,

当220091n -≥,则有312v A A ∈ ,且123,,v v v 两两相识,而

()123123123322009A A A A A A A A A n =+-≥-? .

当3220091n -?≥,则有4123v A A A ∈ ,且1234,,,v v v v 两两相识,而

()123412341234432009A A A A A A A A A A A A n =+-≥-? ,如此继续,

得127,,,v v v 两两相识,而76677111762009i i i i i i A A A A A n ===??=+-≥-? ???

.

当7620091n -?≥,则有781

, i i v A =∈ 且128

,,,v v v 两两相识,而由7620091n -?≥,得6200917

n ?+≥,n 为整数,则1723n ≥. 再说明1723n =是最小的;若1722n =,我们可构造一种情形,使得M 中不存在相互认识的8个人.为此,将2009个人均分为127,,,B B B 等7组,每组287个人,令同组的人互不相识,而异组的任两人皆相识,则M 中任一人v 所认识的人的个数皆为()62871722d v =?=,从M 中任取8个人,必有两个人属于这7组中的同一个组,于是这两人互不相识,因此M 中不存在相互认识的8个人.从而n 的最小值为1723.

6、12个赌徒每日聚赌一次,每次4人一桌,共设三桌;若其中任两人都至少同桌一次,问赌博至少持续了多少天?

解:至少持续了五天.

先说明,5天已够.记12个人为1,2,,12 ,将其两两搭配,记

()1,2a =,()()()()()3,4, 5,6, 7,8, 9,10, 11,12b c d e f =====.先作模式搭配, 安排如下:(一) ab cd ef ; (二) ac be df ; (三) ad bf ce ;

(四) ae bd cf ; (五) af bc de .

即: (一) ()()()1,2,3,4 5,6,7,8 9,10,11,12;

(二) ()()()1,2,5,6 3,4,9,10 7,8,11,12;

(三) ()()()1,2,7,8 3,4,11,12 5,6,9,10;

(四) ()()()1,2,9,10 3,4,7,8 5,6,11,12;

(五) ()()()1,2,11,12 3,4,5,6 7,8,9,10。

其次说明至少需要五天,改记这12人为 1212,,,A A A ,任取一人1A ,

他要与其他11人中的每个人分别同桌,每次同桌都有另外三人作陪,这样至少需要4天,但是11人不能等分成四个“三人组”,于是必有一人,例如2A ,至少两次与1A 同桌,设这两次为:12341256 , A A A A A A A A . 若只安排四天,在后续的两天中,其余6人 7812,,,A A A ,分别要与12,A A 同桌;为使1A 在这两天中能与这6人都同桌,

需将6人等分成两组,每组3人,设这两天1A 所在的桌为:1789A

A A A 与1101112A A A A ,为使2A 在这两天中也能与这6人都同桌,则这两天2A 所在的桌为:2101112A A A A 与2789A A A A ;由于

12个赌徒每天都必需出场一次,于是在这两天中,第三桌的人都是3456A A A A ;但是7812,,,A A A 中至多有三人在第二天与3A 同桌,因此在这四天中,7812,,,A A A 中至少有三人与3A 仍不能同桌,矛盾!因此至少需要五天.

7、若四元集{,,,}E a b c d =中的四个数,,,a b c d 能够分成和相等的两组,则称E 为“平衡集”.试求集{1,2,,100}M = 的平衡子集的个数.

引理:对于给定的正整数k 与集合{1,2,,}N n = ,用(,)f n k 表示不定方程 x y k += (其中 ; ,x y x y N <∈) ……… ① 的解 (,)x y 的个数,

则有 ()1 1 21(,)1 22 1 20 2 1

k k n k f n k k n n k n k n ?-??≤+?????

??-??=---+≤≤-?????

???>-?当 当 当 ……② 证: 当1k n ≤+时,由x y k <<2x x y k ?<+=12k x -?≤12k x -???≤????

, 而当x 取定后,y 的值便唯一确定,故此时()1,2k f n k -??=?

???.

当221n k n +≤≤-时,如在集N 中添加元素1,2,,1n n k ++- ,则可得12k -??????

个解, 但是下列1k n --个解()()()(),1,1,2,2,,1,(1)x y k k k n k k n =-------

中皆含有添加元,不合条件.

于是此时①的解数为 ()()1,12k f n k k n -??=---????

. 又因集N 中的任两数之和21n ≤-,故当2 1 k n >-时, (),0f n k =.引理证完.

回到本题, 对于集M 中的平衡子集 {,,,}E a b c d =, 如果,,,a b c d 中某两数之和等于另两数之和, 则称E 为A -型集;如果,,,a b c d 中某一数等于另三数之和,则称E 为B -型集.

一、先求A -型集的个数(记为()A ?).

(i )当101k ≤,其个数为

()101101

1012111100,115111()1()()222f k k k k k k k k A C

A A ???===-?-?????'''==-==++ ???????????∑∑∑∑∑ 为奇数为偶数 k 为奇数时,记21k m =+,则当k 通过集{}5,6,,101 中的奇数时, m 通过集{}2,3,,50 ,

所以 ()()50

1212m m m A ?=-'=∑. k 为偶数时,记22k m =+,则当k 通过集{}5,6,,101 中的偶数时, m 通过集{}2,3,,49 ,

所以 ()()49

1212m m m A ?=-''=∑. 于是()()()()491112504912m A A A m m ???=?'''=+=-+∑()481

12549j j j ==++?∑ 392001225=+40425=.

()11

. 当102199,k ≤≤其个数为 ()()()()199********,102102111101100222f k k k k k A C k k ?==?-??-?????=

=---- ???????????????

∑∑,

令101,k k '=- 则 1100.k k '-=+ k '通过 {}1,2,,98. 于是

()982115049222k k k A k k ?'=''????

????''=-+-+ ???????????????∑k k ''=+∑∑ 为奇数为偶数

()()22A A .??'''+

当k '为奇数时,令21k m '=+. 当k '跑遍{1,2,,98} 中奇数时,m 跑遍{}0,1,,48 , 所以 48

48

4820011(1)(1)

()(49)(48)222m j j j j j j A m m ?===++'=--==∑∑∑.

当k '为偶数时,令2k m '=. 当k '跑遍{1,2,,98} 中偶数时,m 跑遍{}1,2,,48 ,

48482111(1)

()(50)(49)22m j j j A m m ?==+

''=--=∑∑.

所以,()48

2221

()()()139200j A A A j j ???='''=+=+=∑因A 型集的个数为

12()()()404253920079625A A A ???=+=+=.

二.再求B -型集的个数(记为()B ψ).

对于100k ≤,不定方程x y z k ++=(其中x y z <<,,,x y z M ∈ …… ③

的解数记为()g k . 显然,当5k ≤时,()0,g k =

且由6123, 7124, 8125134=++=++=++=++, 可知

(6)1, (7)1, (8)2g g g ===.

一般有, 当6k ≥,()(3)22k g k g k ??

--=-???? ……… ④

证明如下:对于k 的全体3元分拆 , k x y z x y z =++<< 将其分作两类:

()1.凡是含1的分拆x y z k ++=,其中1x =,将左边诸元,,x y z 各减1,化为

(1)(1)3y z k -+-=-,即3y z k ''+=-形式,归入引理,即②式中的第一情形, 其解数为(3)1222k k --??

??

=-????????.

()2.凡是不含1的分拆 x y z k ++=,其中1x >,将左边诸元各减1,化为

(1)(1)(1)3,x y z k -+-+-=- 即3x y z k '''++=-形式,其中1x y z '''≤<<, 其解数为(3)g k -.

因此 ()(3)22

k g k g k ??=-+-???? , 即④成立. 进而有 6(6)(3)2122k k g k g k +????+-+=-=+????????

⑤ 31(3)()2122k k g k g k ++????+-=-=-????????

⑥ ⑤+⑥ 并注意,对任何k N ∈, 有 1,22k k k +????

+=???????? 所以 (6)()g k g k k +=+ ⑦

利用⑦递推得:,(6)()3(1)n N g k n g k kn n n ?∈+=++- ⑧

于是 2(99)(3616)(3)316316(161)316768g g g =+?=+?+??-=?=,

(100)(4616)(4)416316(161)784g g g =+?=+?+??-=.

从而 ()()()()()()1001001581303991006m m n k B g m g m g g g k n ψ=====

==+++∑∑∑∑ ()()15880331552181n k k g k n k n n ===??=+++-????

∑∑∑()1501552433181n n n n ==+++-????∑

()15

201552641815n n n ==+++=∑15163115161616181525736.62

???+?

+?= 故M 中的平衡子集个数为 ()()7962525736105361A B ?ψ+=+=.

8、设集合(){,M m n =│,m n 互质,}2009,02009m n m n +><<≤,求和: (),1m n M mn

s ∈=

∑. 讲解:由于所给的数值2009较大,于是一般化,将2009换成任一不小于2的整数,k 为此取 (){,,k M m n m n =

互质,},0m n k m n k +><<≤,并记 (),1k k m n M s mn ∈=∑,

分析k s 的变化趋势,易知,

2311111,,12213232s s =

==+=???41111,1423342

s =++=??? , 进而猜想,对于每个2,k ≥都有12k s =. 转而考虑证明,对每个2,k ≥都有10k k s s +-=.

其中 (),1k k m n M s mn ∈=

∑,()1

1,1k k m n M s mn ++∈=∑. 先分解求和区间(和式展布集):

(){,,k M m n m n =互质,},0m n k m n k +><<≤

{}1,0m n k m n k =+=+<<≤ {}1,0*m n k m n k M M '+>+<<≤= ,

(){1,,k M m n m n +=互质,}1,01m n k m n k +>+<<≤+

{}{}1,011,0*m n k m n k m n k m n k M M ''=+>+<<=++>+<<≤= . 于是 ()()1211111 11k k k M M m k

m s s mn mn m k m k m ++'''≤<-=-=-++-∑∑∑∑ ○1 (右端最后一个和式的求和范围是由于 1,m n k m <=+- 则 12k m +<

).由于 ()11221111111k k m m m k m k m k m ++<

∑∑ ○2 当 112k m +≤<时,112

k k m k +<+-≤,则 111222

1111k k k m t k m k k m t m +++<<≤<≤==+-∑∑∑. 且由 1m n k +=+ 以及 (),1m n =知,

12k + 不会被,m n 取到. 从而○2式右端可合并为 111m k k m

≤+∑. 据○1,即有 ()()

111011k k m k m k s s m k m k +≤≤-=-=++∑∑. 于是 1212k k s s s -==== ,因此 200912

s s ==.

9、设X 是一个n 元集,它的2n

个子集组成的集簇记为()E X ,而()F X 是()E X 的非空子集簇,并且F 对于并、交、补运算是封闭的(即,,A B F ?∈则,,,A B A B X A X B -- 也都属于F ).

令k 表示F 中的集的个数,试求k 的取值范围.

解:对于非空的集簇F ,我们定义其中的“素集”如下:如果集簇F 中的一个非空集合A 满足,B F ?∈,则或有A B A = ,或有A B =? ,就称A 是F 中的一个“素集”.

显然,F 中的每个单元素集都是“素集”,若F 中没有单元集,则进而考察其中的2元素集、3元素集、等等. 又若{},F X =?,则X 本身即为F 中的素集.

由于F 中只有有限个集合,故其中的素集也只有有限个,设12,,,m B B B 为

F 中的全部素集,共计m 个,则它们两两不交(否则,如有i j B B B =≠? ,则由,i j B B 的“素性”,可推出i j B B B ==,矛盾.)

再证,1m i i B X == . 即需证,1.m

i

i X B =-=? 事实上,若01m i i X B

B =-= ,而0.B ≠?则一方面,00,,1,2,,i B F B B i m ∈=?= ,

另一方面,由于0B 异于12,,,m B B B ,故0B 不是素集,因此又有某个素集含于它,设0j B B ?,则0,j j B B B =≠? 矛盾,从而1m

i i B X == .

其次说明,F 中的任一个集,都是若干个素集的并.

C F ?∈,则()11

m m

i i i i C C X C B C B ==??=== ??? ,而由i B 的“素性”,每个 i C B 或为空集或等于i B ,即C 为若干个素集的并.反之,据F 的封闭性,每个这类素集之并也是F 的一个元.

因此,F 中的全体集合(包括空集)的个数是

012,1.m m m m m C C C m n +++=≤≤ 即2.m k =其中1.m n ≤≤

再说明,对于1,2,,n 中的每个m ,2m

都可能被k 取到.

将X 中的n 个元任意拆分成m 组,每组至少一个元,即 12,,m i i j X D D D D D D =≠?=? ,则每个i D 都是素集,令F 为由这些“素

集”生成的簇,这时2m k =.故k 的取值范围是122,2,,2n .

10、设M 为n 元集,若M 有k 个不同的子集12,,,k A A A ,满足:对于每个{},1,2,,i j k ∈ ,i j A A ≠? ,求正整数k 的最大值.

解:正整数k 的最大值为12n -.

()01、先证明,存在M 的12n -个子集,两两之交不空;

设{}12,,,n M a a a = ,而1122,,,n A A A - 为集合{}121,,,n a a a - 的全部12n -个子集,令

{}1,1,2,,2n i i n B A a i -== ,则M 的12n -个子集1122,,,n B B B - ,两两之交不空;

()02、再证,对于M 的任何121n -+个子集,其中必有两个子集不相交.

设1122,,,n B B B - 是M 的12n -个不同子集,其中每个皆含n a ;用i B 表示子集i B 在M 中的补集,1(\),1,2,,2n i i B M B i -== ,则对于任意i j ≠,,i j i j B B B B ≠≠,并且j i B B ≠, (因前者含n a 而后者不含),故1122,,,n B B B - ,1122,,,n B B B - 为M 的全部2n 个不同子集,现将上述集合搭配成为12

n -对:()()()11122122,,,,,,n n B B B B B B -- ; 任取M 的121n -+个子集,必有两个子集属于同一对,则这两个子集不相交.

11、在一次晚会上,9位舞星共上演n 个“三人舞”节目,若在这些节目中,任二人都曾合作过一次,且仅合作一次,求n 的值.

解:将9个人看成9个点,若两人同组演出过,则相应两点间连一条线段,于是两两 间都要连线,共得36条线段,而每个三人舞对应于一个三角形,每个三角形有三条边, 当所有的边都出现,且不重复使用,共得36123

n ==个三角形,即12个节目,另一方面, 这样的12个三人舞可以安排,例如:

(1,2,3)(4,5,6)(7,8,9);(1,4,7)(2,5,8)(3,6,9);

(1,5,9)(2,6,7)(3,4,8);(1,6,8)(2,4,9)(3,5,7).

12、如果数集{}12,,,n A a a a = 满足条件:()()1. 3 ; 2. n A ≥中的全体元素可以组成一个等差数列,则称集A 为算术集. 类似地,当集A 的子集B 满足上述条件时,则称B 为A 的算术子集.

试证:对于n 元算术集A ,其算术子集的个数()V n 满足等式:

()()()()()()()2233221V n n T n T T n T n =-+-++-+- .

(其中,()T k 为正整数k 的真因数个数,即小于k 的正因数个数).

证:不妨设,A 中的元素按顺序组成等差数列,则对于其中任一组数

12,,,r i i i a a a ,

它们组成等差数列的充要条件是其下标12,,,r i i i 也组成等差数列.于是我们只要讨论算术集{}01,2,,A n = 的算术子集的个数.

将0A 中以k 为最大元的算术子集个数记为()(), 3w k k n ≤≤,则

()()()()34v n w w w n =+++ ……○

1,显然有 ()()312w T == ......○2, 今证明,一般地有 ()()()()231w k T T T k =+++- (3)

为此,设B 是任一个以k 为最大元的算术子集,若它有2m +个元,由23m +≥,得1m ≥,若该等差数列的公差为d ,得B 的最小元为()1k m d -+.

因 ()11k m d -+≥,得11k m d -+≤,而m 为整数,则11k m d -??≤-????

,从而对于固定的d ,这种子集的个数就是适合111k m d -??≤≤-????的正整数m 的个数,即11k d -??-????

个.今让d 依次取1,2,3,,1k - ,(实际上,仅当12k d -≤时,11k d -??-????才不为0).则 ()111111111231k k k k w k k ?-??-??-??-?????????=-+-+-++- ? ? ? ?????????-????????????????

111231k k k k ---??????=+++??????-??????

,同理,当13k -≥时,又有, ()2221232k k k w k k ---??????-=+++??????-??????

,所以, ()()1w k w k --= 12121212233221k k k k k k k k k k ?--??--??--?-??????????????=-+-++-+ ? ? ???????????????---?????????

??????????? …○4 由于 1, 112 0, 1d k k k d d d k ?---?????-=?????-???????

故○4式右端的和数等于集合{}2,3,,1k - 中属于1k -的因数个数,也即等于集合{}1,2,3,,2k - 中属于1k -的因数个数()1T k -,从而

()()()11w k w k T k --=- ……○

5,据○2及○5立即得到

()()()()231, 3 w k T T T k k n =+++-≤≤ ○

6,由○1、○6得 ()()()()()()()2233221V n n T n T T n T n =-+-++-+- .

13、对于2n 元集合{}1,2,,2M n = ,若n 元集{}12,,,n A a a a = ,

{}12,,,n B b b b = 满足:,A B M A B ==? ,

且11n n

k k k k a b ===∑∑,则称A B 是集M 的一个“等和划分”(A B 与B A 算是同一个划分).

试确定集{}1,2,,12M = 共有多少个“等和划分”.

解一:不妨设12A ∈,由于当集A 确定后,集B 便唯一确定,故只须考虑集A 的个数,设{}126,,,A a a a = ,6a 为最大数,由121278+++= ,则

12639a a a +++= ,612a =,于是 1234527a a a a a ++++=,

故{}112345,,,,A a a a a a =中有奇数个奇数.

()1、若1A 中有5个奇数,因M 中的六个奇数之和为36,而27369=-,则

{}11,3,5,7,11A =,这时得到唯一的{}1,3,5,7,11,12A =;

()2、若1A 中有3个奇数、两个偶数;用p 表示1A 中这两个偶数12,x x 之和;q 表示1A 中这三个奇数123,,y y y 之和,则6,9p q ≥≥,于是21,18q p ≤≤.共得1A 的24种情形.

其中,()

01、当6,21p q ==,则()()12,2,4x x =,()()()123,,1,9,11,3,7,11y y y =, ()5,7,9;可搭配成1A 的3个情形;

()0

2、当8,19p q ==,则()()12,2,6x x =,()()()()123

,,1,7,11,3,5,11,3,7,9y y y =;可搭配成1A 的3个情形;

()0

3、当10,17p q ==,则()()()12,2,8,4,6x x =,()()()123

,,1,5,11,1,7,9y y y =, ()3,5,9,可搭配成1A 的6个情形;

()0

4、当12,15p q ==,则()()()12,2,10,4,8x x =,()()()123

,,1,3,11,1,5,9y y y =,()3,5,7,可搭配成1A 的6个情形;

()0

5、当14,13p q ==,则()()()12,4,10,6,8x x =,()()()123

,,1,3,9,1,5,7y y y =,可

搭配成1A 的4个情形;

()0

6、当16,11p q ==,则()()12,6,10x x =,()()123

,,1,3,7y y y =; 可搭配成1A 的1个情形;

()0

7、当18,9p q ==,则()()12,8,10x x =,()()123

,,1,3,5y y y =; 可搭配成1A 的1个情形.

()3、若1A 中有一个奇数、四个偶数,由于M 中除12外,其余的五个偶数和24681030++++=,从中去掉一个偶数,补加一个奇数,使1A 中五数之和为27,分别得到1A 的4个情形:()()()()7,2,4,6,8,5,2,4,6,10,3,2,4,8,10,1,2,6,8,10.

综合以上三步讨论,可知集A 有124429++=种情形,即M 有29种“等和划分”. 解二:元素交换法,显然

661139i i i i a b ====∑∑,恒设12A ∈; ()01、首先注意极端情况的一个分划:{}{}00

1,2,3,10,11,12,4,5,6,7,8,9A B ==,显然数组{}1,2,3与{}10,11,12中,若有一组数全在A 中,则另一组数必全在A 中; 以下考虑10,11两数至少一个不在A 中的情况,为此,考虑00,A B 中个数相同且和数相等的元素交换:

()0

2、()()()10,15,6,4,7?;()()()10,25,7,4,8?;()()()()10,36,7,5,8,4,9?; ()()10,2,34,5,6?;共得到8个对换;

()0

3、()()()11,15,7,4,8?;()()()()11,26,7,5,8,4,9?;()()()11,36,8,5,9?; ()()11,1,34,5,6?;()()11,2,34,5,7?;共得到9个对换;

()0

4、()()()10,11,16,7,9,5,8,9?;()()10,11,26,8,9?;()()10,11,37,8,9?; ()()()10,11,1,24,5,7,8,4,5,6,9?;()()()10,11,1,34,6,7,8,4,5,7,9?;

()()()()10,11,2,35,6,7,8,4,6,7,9,4,5,8,9?;共得到11个对换.每个对换都得到一个新的划分,因此,本题共得1891129+++=种等和划分.

14、设正整数a 的各位数字全由1和2组成,由其中任意() 2k k ≥个连续数位上的数

字所组成的k 位数,称为数a 的一个“k 段”. 若数a 的任两个“k 段”都不相同,证明:对于具有这种性质的最大正整数a ,其开初的一个“1k -段”和最后的一个“1k -段”必定相同. 证:设12n a x x x = 是一个具有这种性质的最大正整数,由a 的最大性,在其后面无论添加1或2,所得到的1n +位数1121n a x x x = 以及2122n a x x x = 中,都有两个相同的“k 段”. 设在1a 中有 1121i i i k n k n x x x x x ++--+= ;在2a 中有

1122j j j k n k n x x x x x ++--+= . 显然i j ≠,

(因11i k j k x x +-+-≠),且11i n k ≤≤-+, 11j n k ≤≤-+,如果1i =或1j =,则直接去掉相应“k 段”中的末位数,可知结论成立;如果2i ≥且2j ≥,因12212i i i k n k n j j j k x x x x x x x x ++--+++-== ,考虑

各自的前一位数字111, , i j n k x x x ---+,它们只取1和2两个值,其中必有两数相同,于是数a 中有两个相同的“k 段”,矛盾. 因此,i j 中必有一个为1,故结论得证.

15、设3n ≥,问是否能将集合{}1,2,,3M n = 分拆成三个这样的n 元子集123,,A A A ,使得对于每个子集i A ,其中任两个数之和被31n +除得的余数,既不为零,也不等于该集合中的另一个数?

试就 3, 4, 5n =分别讨论上述问题.

解:()

01.3n =时,对于集合{}1,2,,9M = ,存在这种分划, 例如分成 {}{}{}11,2,45,6,73,8,9T = ; {}{}{}21,2,73,4,56,8,9;T =

{}{}{}31,2,46,7,83,5,9;T = {}{}{}41,5,72,3,46,8,9;T =

{}{}{}51,2,43,6,85,7,9;T = {}{}{}61,3,52,4,76,8,9;T =

{}{}{}71,2,53,6,84,7,9;T = {}{}{}81,3,62,4,75,8,9;T =

{}{}{}91,2,64,5,73,8,9;T = {}{}{}101,2,73,5,64,8,9;T =

{}{}{}111,2,63,4,85,7,9;T = {}{}{}121,3,52,6,74,8,9;T =

{}{}{}131,2,64,7,83,5,9;T = {}{}{}141,5,72,3,64,8,9;T =

{}{}{}151,3,54,7,82,6,9;T = {}{}{}161,4,82,3,65,7,9;T =

{}{}{}171,3,56,7,82,4,9;T = {}{}{}181,6,82,3,45,7,9;T =

{}{}{}191,3,64,7,82,5,9;T = {}{}{}201,5,82,3,64,7,9;T =

{}{}{}211,4,82,6,73,5,9;T = {}{}{}221,5,73,4,82,6,9;T =

{}{}{}231,5,73,6,82,4,9;T = {}{}{}241,6,82,4,73,5,9;T =

(它已穷尽所有分法,其中分法21k T -与2k T 对偶; 即,若将一个式子中的每个元素a 换为 10a -,便得另一式.)

()0

2.4n =时,对于集合{}1,2,,12M = ,存在这种分划,例如分成 {}1,2,,12= {}{}{}1,2,4,75,9,10,113,6,8,12 ,

{}1,2,,12= {}{}{}1,5,7,102,3,4,86,9,11,12

(只有这一对分法;若将一个式子中的每个元素a 换为13a -,便得另一式.) ()0

3.5n =时,对于集合{}1,2,,15M = ,不存在这样的分划. 为此,考虑适合条件的所有5元子集,称题中条件为性质P ,若5元子集{}12345,,,,A a a a a a P =∈ ,令16, 1,,5i i b a i =-= ,则{}12345,,,,B b b b b b P =∈, 称这种子集,A B 为“相伴集”,显然,这两种子集,A B 一一对应. 为此,考虑数对 ()11,15,α=()()()()()()2345672,14,3,13,4,12,5,11,6,10,7,9,

αααααα======以及8,由于每对数的和为16,故不能同在一集,记含1的五元子集类为1F ,含

15的五元子集类为1F -,其余五元子集类为0F ,显然,1F 的集与1F -的集一一对应,且1F 的集与1F -的集中都不能含有相邻的数。今求1F 中的所有集合:

为此, 先证明,1F 类中的任一集合不含元素5,7,13. 采用反证:

1F 类中如存在集合A ,使5A ∈,因为14,6,11,12A A ∈??.(这是由于4,6皆与5相邻,且()()1150mod16,1251mod16+≡+≡,以下的同余记号中均省略mod16)),于是另三数将取自()()()2,14,3,13,7,9,8,10,且()2,14与()3,13中的任两数不能共存,这是由于,若取2,因325,1352,+=+≡则3,13皆不能取;若取14,因1453,14113,+≡=+则3,13也不能取。反之,如取3或13,据以上四式知,2与14也不能取。

()1.若三数取自()()3,13,7,9,8,10,若7A ∈,则8,9,10A ?因718+=,790,7101+≡+≡,故7不能取,因此另三数将取自()3,13,8,9,10,显然9A ?,否则不能取8,10,也不能同取3,13,矛盾. 于是A 中的数是()1,3,5,8,10,或()1,5,8,10,13,但前组中有358+=,后组中有5813+=,矛盾。

()2.若三数取自()()2,14,7,9,8,10,同样知7不能取,即另三数在()2,14,8,9,10中选取,进而知9不能取,否则不能取8,10,也不能同取()2,14.于是A 中的数为()1,2,5,8,10或()1,5,8,10,14,而前组中有2810+=,后组中有10148+≡,矛盾。所以5A ?.

1F 类中如存在集合A ,

使7A ∈,因为16,8,9,10A A ∈??,(6,8与7相邻,790,+≡ 7101+≡),因此另三数将取自()()()2,14,3,13,4,12,11,且由 347,+=1273+≡, 1341,13121+≡=+知,()()3,13,4,12中的任两数不能并存,

()1. 若另三数取自()()2,14,3,13,11,则11313142A A A A A ∈???∈???∈,故A 中的数是()1,2,7,11,13,而其中有21113+=,矛盾.

()2. 若另三数取自()()2,14,4,12,11,则11124,A A A ∈???∈而7411+=,矛盾. 因此7.A ?.

1F 类中如存在集合A ,使13A ∈,因为13,4,12,14A A ∈??,(3130,+≡4131+≡,

12113,14113+==+),且据前述,5,7A ?,因此另三数将取自()6,10,2,8,9,11, 若取9,则6,8,10A ?(因8,10与9相邻,而9136+≡),故A 中的数只有1,2,9,11,13,而其中2911+=,矛盾.

故9不取,A 中另三数取自()6,10,2,8,11因21113+=,故2,11不能同取,8必取,且

()6,10中必取一数,若取2,则6,10不能取,

(因628,2810+=+=),矛盾,若取11,因 11138+≡,也得矛盾.故13A ?,从而5,7,13A ?.

所以A 中除1外的另四数取自()()()()2,14,4,12,6,10,8,9,3,11,(括号中的每对数不能同取),若取11,则6,10,12不能取(因10,12与11相邻,6111+≡),余下的数为()2,14,

()()3,4,8,9,每组恰取一数,且3不能取(否则2,4,8,14皆不能取)

,于是得两组解:

()()1,2,4,8,11,1,4,8,11,14;

若不取11,如果取3,则2,4,14不能取(因2,4与3相邻,1431+≡),余下三数从()()6,10,8,9,12中取出,每组恰取一数,得两组解:()1,3,6,8,12,()1,3,8,10,12; 如果11与3都不取,即要从()()()()2,14,4,12,6,10,8,9中取出四数,每组恰取一数,则只有一解:()1,6,9,12,14.

(这是由于,若有8,A ∈ 如果26,10A A ∈??,矛盾;如果

14610412A A A A A ∈???∈???∈,因12141010A +≡??,矛盾。

故8A ?9106A A A ?∈???∈,如果2,A ∈因426,1262+=+≡,则4

,12A ?,矛盾,故214A A ??∈,因1464412A A +≡???∈,故得一解 ()1,6,9,12,14) 由此知,1F 类共有五个集:()11

,2,4,8,11,A =()()231,4,8,11,14,1,3,6,8,12,A A == ()()451,3,8,10,12,1,6,9,12,14A A ==;据相伴性得,1F -类也恰有五个集:()()()12315,14,12,8,5,15,12,8,5,2,15,13,10,8,4B B B ===,

()()4515,13,8,6,4,15,10,7,4,2B B ==.

用k C 表示0F 类集合,考虑集M 的分划:i j k M A B C = ,注意其中,i j A B 中不能同时含有8及任何相同元素,这种搭配只有三种:()()()355355,,,,,A B A B A B ,对于这三种搭配,分别有,(){}1355,9,11,13,14C M A B =-= ,(){}2532,3,5,7,11C M A B =-= , (){}3553,5,8,11,13C M A B =-= ;显然,1C 中有5914+=,2C 中有235+=,3C 中有358+=,即123,,C C C 都不具有性质P ,因此,不存在合于题意的分划.

15、奥运会排球预选赛有n 支球队参加,其中每两队比赛一场,每场比赛必决出胜负,如果其中有k (3k n ≤≤)支球队12,,,k A A A ,满足:1A 胜2A ,2A 胜3A ,…,1k A -胜k A ,k A 胜1A ,则称这k 支球队组成一个k 阶连环套;

证明:若全部n 支球队组成一个n 阶连环套,则对于每个k (3k n ≤≤)及每支球队i A ()1i n ≤≤,i A 必另外某些球队组成一个k 阶连环套.

证明:以12,,,k A A A 为顶点,如球队i A 胜j A ,则在两点间连一有向边:i j A A →,如此得n 阶竞赛图G .据条件,G 的n 个顶点可以排成一个n 阶有向圈,设为: 121n A A A A →→→→ ,于是G 的任两点可沿箭头方向相互到达.

先证明,任一球队i A 必在某个三阶连环套中,用,i i S T 分别表示被i A 击败了的球队集合和击败了i A 的所有球队集合,由于G 双向连通,必有,j i k i A S A T ∈∈,使j k A A →,于是,,i j k A A A 组成三阶连环套;

假若已证得,对于()3k k n ≤<,图中存在以i A 为一顶点的k 阶连环套

()121i k C A A A A A = ,圈C 之外的点的集合为M ;

若M 中有一点P ,它所表示的球队既击败了圈C 中的某个队k A ,又被圈C 中的另一个队j A 所击败,点,k j A A 把圈C 分成两条有向路12,C C ,其中一条,

例如1C ,它与有向路j k A P A →→组成有向圈,如图所示.

依次考虑路2C :12,,,,j j j k A A A A ++ 上各点与点P 间的邻接情况,必有相邻的两点1,j r j r A A +++,满足j r A P +→而1j r P A ++→,今去掉边1j r j r A A +++→,而将路1j r j r A P A +++→→插入其间,便得到一个含有顶点i A 的1k +阶连环套;

若M 中的任一点P ,它所表示的球队要么击败了圈C 中的每个队,要么被圈C 中的每个 队所击败,则集M 可分为两个不交的子集S 与T ,其中S 中的任一队,战胜了圈C 中所有的队,而T 中的任一队,负于圈C 中所有的队;由于图G 双向连通,故在集T 中必有点u ,集S 有点v ,使v u →,今在圈C 中任意去掉一个点j A ,()

j i A A ≠,而用路v u →代替,便得到一个含有顶点i A 的1k +阶连环套;故结论对于1k +成立,由归纳法,结论成立.

16、矩形玻璃台板碎裂成一些小玻璃片,每块碎片都是凸多边形,将其重新粘合成原矩形后,有交结点30个,其中20个点在原矩形的周界上(包括原矩形的四个顶点),其余10点在矩形内部.在矩形的内部有44条粘缝(两个结点之间的线段算是一条粘缝,如图所示). 试求该矩形台板所碎裂成的每种类型(指三角形、四边形、五边形等等)的碎片块数. (若凸多边形的周界上有n 个点,就将其算作n 边形).

解:设全部碎片中,共有三角形3a 个,四边形4a 个,

… …,k 边形k a 个(34,,,k a a a 为非负整数).

一个多边形,若其周界上有n 个顶点,就将其看成是n 边形,

j

(例如图中的多边形ABCDE 要看成五边形).于是这些多边形的内角和S 角为: S 角()3422k a a a k πππ=?+?++?- .

从另一方面看,矩形内部有10个结点,对于每个点,围绕它的多边形顶角和为2,π 10个内结点共获得102π?弧度;矩形边界线段内共有16个结点,在每个这种结点处,各多边形的顶角在此汇合成一个平角,16个这种结点共获得16π弧度;而原矩形的四个顶点处,共收集到多边形碎片的2π弧度.因此,

S 角2016238ππππ=++=,于是,()342238k a a k a +++-= ……○

1. 再计算边数和S 边:由于每个n 边形有n 条边,则

S 边3434k a a ka =+++ .

另一方面,在矩形的内部的44条粘缝,每条都是两个多边形的公共边,故都计算了两遍,矩形周界上的20条线段各被计算了一遍,因此,S 边24420108=?+=,则

3434108k a a ka +++= ……○

2. ○

2-○1得,()34270k a a a +++= ,因此碎片的块数为: 3435k a a a +++= ……○

3. ○

1—○3得,()4562333k a a a k a ++++-= …… ○4. 注意所有i a 为非负整数,故780k a a a ==== ,456233a a a ++= ……○5. 由○5得,4563, 0a a a ===;或者4561, 0a a a ===;或者4560, 1a a a ===……○6. 因此由○3、○5、○6得,本题的解共有三种情况:

即全部碎片总共有35块;其中,或者含有32个三角形,3个四边形;或者含有33个三角形,1个四边形,1个五边形;或者含有34个三角形,1个六边形.

17、对于周长为n *

()n N ∈的圆,称满足如下条件的最小的正整数n P 为“圆剖分数”

: 如果在圆周上有n P 个点12,,,n p A A A ,对于1,2,,1n - 中的每一个整数m ,都存在

两个点 ,i j A A (1,)n i j P ≤≤,以i j A A 和为端点的一条弧长等于m ;圆周上每相邻两点间的弧长顺次构成的序列12(,,,)n n P T a a a = 称为“圆剖分序列”.例如:当13n =时,圆剖

分数为134P =,如图所示,图中所标数字为相邻两点之间的弧长.

相关主题
文本预览
相关文档 最新文档